[Boards: 3 / a / aco / adv / an / asp / b / bant / biz / c / can / cgl / ck / cm / co / cock / d / diy / e / fa / fap / fit / fitlit / g / gd / gif / h / hc / his / hm / hr / i / ic / int / jp / k / lgbt / lit / m / mlp / mlpol / mo / mtv / mu / n / news / o / out / outsoc / p / po / pol / qa / qst / r / r9k / s / s4s / sci / soc / sp / spa / t / tg / toy / trash / trv / tv / u / v / vg / vint / vip / vp / vr / w / wg / wsg / wsr / x / y ] [Search | Free Show | Home]

The hard problem of consciousness

This is a blue board which means that it's for everybody (Safe For Work content only). If you see any adult content, please report it.

Thread replies: 305
Thread images: 25

File: chalmers.jpg (90KB, 828x712px) Image search: [Google]
chalmers.jpg
90KB, 828x712px
What is /sci/'s response to the hard problem of consciousness? Why is there something "that it is like" to be a brain? Why doesn't all the data processing go on "in the dark", without the phenomenal experience? Do you see a big explanatory gap between function and experience here?

http://www.strawpoll.me/12358244

https://en.wikipedia.org/wiki/Hard_problem_of_consciousness
http://www.iep.utm.edu/hard-con/
>>
>>8682044
Well at this point, I don't think science has much to say on this problem, which remains tied to philosophy. I'm sure there's an evolutionary reason why we have conscious experiences, but the reason why is beyond anything I'm aware of.

I definitely think this is a serious question, and I imagine steps towards its solution would be the development of a better understanding of self-reference.
>>
>>8682044
>consciousness?
Infinite minds imagining the limited concepts of space and time. It's bound to be confusing.
>>
The "hard problem" is only a problem in the substance dualism model of consciousness. There are models that account for what humans perceive without having so many issues. That problem is arguably a paradox even, which would mean it falsifies dualism.
>>
>>8683202
>The "hard problem" is only a problem in the substance dualism model of consciousness
Absolutely not. You don't need to be a dualist to acknowledge that qualia are very different entities than the physical stimuli that cause them. It's quite obvious that 'redness' as it is experienced is not equivalent to a set of electromagnetic wavelengths. The experience of 'red' is produced by the brain/mind from the stimuli. In some sense, this is comparable to comparing sound as shifts in air pressure with language. From the basic stimuli, we abstract a complex semiotic system.
>>
I guess consciousness is just a powerfull data processing and decision making algorithm. I would say it arises because there are a lot of complex auto-regulating loops in our brain.

Language might also help in doing abstraction
>>
It's interesting to note IMO that we can even talk about "consciousness", and that we all know what is meant by the word. This suggests that somehow, the "what it is like to be conscious" is accessible in the physical world (i.e. by the brain). I think this suggests that it should be possible, at least in principle, to solve the hard problem.
>>
>>8684419
>It's quite obvious that 'redness' as it is experienced is not equivalent to a set of electromagnetic wavelengths.
But it is. You just think it's not because language isn't perfectly communicative and humans aren't perfect at imagining. A robot can acquire knowledge of red without ever being exposed to it by just giving it the wavelength of light that produces red.
>>
>>8682044
>qualia are very different entities than the physical stimuli that cause them.
We're aware of the physical properties and reactions that cause us to experience most physical stimuli in a predictable fashion, so I wouldn't say it's VERY different. While "Red" doesn't exist in nature, the nature of color is well known -- now, if everyone was experiencing different colors then that would be truly perplexing.

Most of the "why" questions are superfluous. Like any other organism, our only essential purpose is to survive and procreate, and I don't think there's any reason to believe that the qualia are anything but evolutionary developments. I think the more developed our brains become, the more we have a tendency to make mountains out of molehills and complicate the fuck out of everything.

Consciousness is essentially atoms interacting and trying to understand each other (in mysterious ways) -- that's good enough for me.
>>
>>8682044
>>>/x/
>>
>>8684805
It also creates big holes in dualist arguments, because of the interaction problem (if experience is separate from the physical are separate, how can they interact?) Epiphenomenalism solves the interaction problem by saying phenomenal experience doesn't affect the physical, yet the very thing you describe seems to indicate just that.

A possible solution to that is if the physical can epiphenomenally cause the mental, then wouldn't it be possible that the physical would process information that could in some sense describe features of the mental, explaining how the physical can come up with verbal reports/thoughts about it: It's simply corresponding with the phenomenal experience without directly accessing it. An analogy would be: when a computer burns a disc, the would-be contents of the disc is going to be in the computers memory when the disc is created, and if saved, the computer could describe the disc, without reading anything from it. The computer has then caused the disc, and is able to describe it without reading anything from its final form. This leaves the question of how exactly information crosses to the "phenomenal realm", but that gap doesn't create as many problems as causal interaction the other way around.

To expand on this gap, you could ask, how is it that something as irreducible and and seemingly fundamental as phenomenal experience can directly interact with something as high level as neurobiology? This is where maybe the integrated information theory can come into play. Phenomenal experience is not directly caused on a level where matter interacts very fundamentally, but on the higher levels where information is very integrated, and from that experience is formed through some kind of irreducible emergence.
>>
>>8685009
Fuckin hell, this truly is the NON-problem of consciousness.

This explains it very succinctly without the fluff: https://en.wikipedia.org/wiki/Popper's_three_worlds
>>
>>8685009
How does this solve the P-zombie problem?
>>
File: GodfreyKneller-IsaacNewton-1689.jpg (198KB, 407x559px) Image search: [Google]
GodfreyKneller-IsaacNewton-1689.jpg
198KB, 407x559px
>Hard problem doubters
It happens because of information processing and then magic

>Hard problem believers
It happens because of some phenomenon we don't understand yet
>>
>>8685046
You got that backwards friendo.
>>
>>8685051
You don't understand the hard problem. Dan dennet is a quack
>>
>>8685042
Not sure what you mean by solve, but it's consistent with the assumptions the p-zombie argument makes; that subjective experience has no causal powers of its own.
>>
ITT: People who cannot even grasp the concept of irreducibility
>>
>>8685053
The problem is simple: somebody misinterprets their observations then makes a bunch of assumptions based on that, which they then label as "obvious facts".

The hard problem claims a bunch of properties for consciousness that I do not observe it as having. And since my observations do not create the same contradictions as the ones pointed out in that article, my observations are necessarily more reliable.
>>
File: download.jpg (8KB, 233x216px) Image search: [Google]
download.jpg
8KB, 233x216px
I believe that not every human is truly conscious.
>>
>>8685074
If you've been color blind your entire life, and other people claim they can see color, would you assume that your observations are necessarily more reliable because you don't misinterpret your observations with silly color illusions?
>>
>>8685071
If a P-zombie is truly indistinguishable, then it disproves qualia by showing that mental properties are unnecessary. If a P-zombie is distinguishable, then it disproves qualia by showing that mental properties are physical.
>>
>>8684899
I'm skeptical about this given how processed stimuli are once you actually experience them. I think an interesting thing to consider is the so-called McGurk effect, on which I encourage to read. It's quite striking that the sounds you perceive are not systematically determined by the wavelengths your ears pick up. I think qualia, such as colors or sounds, are more of an abstraction created by the mind than faithful representations of the stimuli to which we are exposed.

https://en.wikipedia.org/wiki/McGurk_effect
>>
File: 1483478168375.jpg (68KB, 751x600px) Image search: [Google]
1483478168375.jpg
68KB, 751x600px
I could build a range of information processing systems, starting with simple ones, and increasing the complexity up to human level.

At some level of complexity in this series of systems, I will have built a machine which is conscious.

I will know every component in this machine, but as for explaining why it is conscious, I will only be able to point to the differences between the conscious machine, and the simpler, unconscious machines.

I will have no better understanding of consciousness besides that this machine has some component/module that the other machines lack.
>>
>>8685084
>And since my observations do not create the same contradictions
>>
>>8685092
But, a P-zombie being indistinguishable or not doesn't necessarily say something about the the realness of qualia, it says something about the realness of its causal effects.

There is also a difference between functionally indistinguishable and intrinsically indistinguishable. Two things can appear to function identically, yet don't at some level we can't access empirically.
>>
>>8685097
I don't see how that has any relation. Human perception is obviously not "pure" because the brain takes various shortcuts, but that has nothing to do with qualia.
>>
>>8685107
Realness = Causal Effects

If something had no causal effect, nobody would ever know about it.
>>
>>8685097
>I think qualia, such as colors or sounds, are more of an abstraction created by the mind than faithful representations of the stimuli to which we are exposed
I think you're cherrypicking with anecdotal evidence (McGirk effect). If people without identifiable mental impairments were experiencing the McGirk effect in their daily lives, and as a result, there was no consensus on basic laws and properties, you might have a case. The subjective assessment of these laws and properties is irrelevant.
>>
>>8685125
It's not inconceivable to assume that some aspects of the universe has no observable casual effects. If there is, I don't think it's fair to say they are any less real.
>>
>>8685098
What is the name of this fallacy?
>>
>>8685031
>Popperian cosmology rejects this essentialism, but maintains the common sense view that physical and mental states exist, and they interact.
Not much of an explanation there.
>>
>>8685177
Sure it is. Only dopey philosophers think anything else actually matters.

Our consciousness didn't arise from nothing, as there is a physical process that gives rise to our ontology. Our experience is based on physical processes, and I don't think the subjective feelings that arise when observing these processes (qualia) should be relevant to anyone but artists and writers.

Different colors may produce different feelings, but regardless of the shade, the vast majority of people will be able to identify the color. Can we perfectly describe it? The artists and writers will try... and fail.
>>
>>8685159
It's not a fallacy
>>
>>8685260
It's assuming the outcome of an experiment that has never been performed.
>>
>>8685267
I believe it's called a "thought experiment"
>>
>>8685141
The McGurk effect is a very solid effect which has nothing to do with mental illness. In fact, I encourage you to watch stimuli which incite the effect. Also, any study of the mind is hopeless if we don't take into consideration subjective assessments. If your hope was to conduct cognitive science in the way the physical sciences are done, I doubt you would get many interesting results. Behaviorism, while not completely effaced, is not particularly relevant to much of the progress which has been done in understanding cognitive systems.
>>
>>8685245
>I don't care about it
>therefore the problem is solved
>>
None of us truly exist, we're all just delusions.
Your whole life, personality and being just a dream.
A movie projection playing inside your head, only there's no one watching.

The delusion of being a person arose out of convenience, to spread your chemicals, there's no one there being deluded.
There's no unity or experience of being a person, it's all just disconnected sensory spikes, the brain constructs the delusion as a dream in hindsight.
>>
>>8685292
>The delusion of being a person arose out of convenience
That's not what consciousness is.
>>
File: 1478673820810s.jpg (5KB, 250x191px) Image search: [Google]
1478673820810s.jpg
5KB, 250x191px
>>8685046
>>
>>8685292
Read about emergence before you share your 2 cent philosophy.
>>
File: 220px-Daniel_dennett_Oct2008.jpg (5KB, 220x156px) Image search: [Google]
220px-Daniel_dennett_Oct2008.jpg
5KB, 220x156px
ITT: We post attention whoring brainlets
>>
>>8685276
>the McGurk effect is a very solid effect which has nothing to do with mental illness
I realize that. I'm saying that there's a reason it's called an effect, and it's simply a matter of the brain adapting to stimuli and filtering the most relevant information for optimal performance. Also, I should have said *were/weren't experiencing this effect.*

>Behaviorism, while not completely effaced, is not particularly relevant to much of the progress which has been done in understanding cognitive systems
At this point, progress is extremely relative.
>>
>>8685282
You're welcome. Screencap this thread.
>>
>>8685321
Can you retards stop arguing about mcgurk effect? It has nothing to do with the discussion of consciousness. McGurk effect is an inductive bias used by the brain for processing of speech.
>>
>>8685335
The discussion on the McGurk effect was brought up to discuss the link between qualia and the stimuli which cause them. The link between the perception of qualia and consciousness is salient, and I don't see how you're missing this point.
>>
>>8685335
The other guy started it. And don't you ever tell me when I can and cannot discuss McGurk.
>>
>>8685346
>>8685338
I think we have more of a case of Dunning-Kruger effect going on with you two.
>>
>>8685351
Oh yeah? Well you're just a pseudo-intellectual.

Alright, I'm done shitposting. Kek
>>
>>8685351
Alright, I've got to admit that's pretty clever of you. Good job.
>>
File: dan_dennett.png (39KB, 800x600px) Image search: [Google]
dan_dennett.png
39KB, 800x600px
>>8685053
Related meme.

>>8685079
Really makes you wonder about the nature of some of the people in this thread.

>>8685292
>You're not having experiences, you're just experiencing the illusion that you're exper... wait.
>>
>>8684913
>- now, if everyone was experiencing different colors then that would be truly perplexing.
Is it proof that everyone experiences a similar red/blue/green/colors by using color theory as evidence?
I "know" you and I experience a similar red and yellow because color theory says those colors are sharp and contrasting and I find that combination sharp and contrasting but that wouldn't be the case if my "red and yellow" was something more like your "brown and orange"
>>
What's to say that there is only 1 stream of subjective experience in your brain? What if you're only one of the many systems that have experiences, and the actions you consider subconscious is still subjectively experienced in another part of your brain? That would mean that there are many conscious entities in your brain that are all experiencing reading this text in one way or another, depending on what functions that part of the brain is concerned with. Suicide then becomes a very direct ethical concern considering a single conscious entity is not in charge of making every decision.

This idea begs a question of identity. The thoughts and actions of your person would therefore not longer be you. The only thing that is truly yours is your conscious experience, which is arguably still true even if there is only 1 stream of consciousness, but this illustrates that reality in a clearer way.
>>
>>8686194
The idea of subjective experience is completely unfounded.
>>
>>8686102
Because people can identify hundreds of shades. And if people were experiencing different combinations of colors, there would not be consensus on various shades of color.
>>
>>8686205
>The idea of subjective experience is completely unfounded.

this may be the most laughable thing I've ever read on /sci/. Do you have any idea what you actually meant by that or were you just stringing words together

Subjective experience is quite literally the only thing in the universe which is founded.

Everything you experience could be a complete illusion... but you are still experiencing something.
>>
>>8686286
The thing that we cannot imagine the world without; the thing that we consider "essential" is the acting force and not a passive observer. The statement isn't "I observe, therefore I am" It's "I think, therefore I am". Though "I" is not really good there.
>>
File: 1413795448828.jpg (19KB, 550x550px) Image search: [Google]
1413795448828.jpg
19KB, 550x550px
>>8685276

>McGurk effect

That's fascinating as fuck
>>
>>8682044
theres no proof that anyone but me is conscious
>>
>>8686496
That's true, but it is a reasonable assumption.
>>
>>8686496
This line of thinking really irritates me. I mean, like >>8686974 said, it's true, but people who use it in these conversations should be gassed.

It reminds me of that silly question about the tree falling in the forest: does it make a sound? It's safe to assume, beyond a reasonable doubt, that the tree would still create the physical reaction that we would perceive as sound, but if there's no one around to process the stimuli... then there's no one to perceive it as sound.

It's safe to assume, beyond a reasonable doubt, that the fate of the universe doesn't hinge on our ability to perceive it, and that matter and energy would continue to interact in the same fashion without us.

Yes, this is empiricist, materialist, physicalist, whatever you wanna call it.
>but like... what is reality, maaaan

t. Someone who thinks the simulation hypothesis is plausible.
>>
>>8687033
I agree with you, but I think there is a lot of nuance you can add to an idealistic viewpoint, to the point where whether or not it's really idealism is a bit murky.

Let's assume that there is some sort of objective reality, that function even if you and I were not here experiencing it. That doesn't mean that the reality that we experience, isn't a complete fabrication of our minds. The fabrication uses inputs from the objective reality, but what we're experiencing isn't at all representative of the true objective qualities of this objective reality. The sound of the tree, how the tree looks, and what we think of trees, are not qualities of the objective reality, but of the reality that we create. With all that in mind, the notion that the tree didn't make a sound might not be as silly as it first appears, considering the fact that neither sound, "falling", or "ground", can be intrinsic qualities of the objective reality.
>>
>>8687121
I think we're on the same page.
>>
There's no proof that anybody but you si conscious. Therefore: the only correct conclusion is that you are the only thing that truly exists, and everything around you is just a Show you put on for yourself, like a dream.
>>
>>8687329
That's some solid logic right there lad. No wonder postmerdernism has such a strong influence on scientific research.
>>
Anyone else find it peculiar how hard of a line there is between people who can conceive of a hard problem, and those who can't? Those who argue for the hard problem are amazed that the people on the other side can't understand it, and the people arguing against a hard problem are amazed that people make such a big deal out of nothing.

Is it that language about mental states is so private that words have vastly different definitions depending on who you talk to, or do people actually understand the gap between experience and function, but still don't think it needs to be ontologically explained? People who don't see a hard problem often don't directly respond to this question, so it would be interesting if someone here did.
>>
>>8688145
It's because the definition of consciousness is either never made clear, a nonsense definition is given, or a definition that doesn't even work within the problem is given.
>>
>>8688151
The definition of consciousness when talking about the hard problem is always stated clearly: we're talking about the subjective experience, what "it is like" to be you, the fact that you are subject to experiences. You should, intuitively be able to understand what's meant by this.

Is this what you would consider a nonsense definition? If so, then understand that it's not a definition meant to explain what consciousness is - it's meant to refer to the very phenomena that we're trying to understand. If people knew what it was, there wouldn't be a hard problem.
>>
>>8688168
The problem is that definition makes false assumptions. It's nowhere near as "primal" as people think it is.
>>
>>8688191
Would you mind elaborating a little bit?
>>
>>8682044
>What is /sci/'s response to the hard problem of consciousness?
Monads. That guy Leibniz was right.
>>
>>8688196
Well I'm glad this huge philosophical puzzle was solved by no more than six words.
>>
>>8688191
What false assumptions does it make and what do you mean exactly by primal?

I would argue that your experience is the most primal thing there is, and the only thing you can be truly sure exists, for the same reasons Descartes did.
>>
>>8688192
Conscious motions are distinguished from non-conscious motions by the presence of X. Consciousness is the conceptualization of X.

So the question is "what is X". All this stuff about "experience" is just a huge red herring.
>>
>>8688205
Now you're talking about consciousness in the form of the cognitive functional systems of the brain. The question of what the difference is between subconscious and conscious actions is a question that can be solved by a functional explanation of the brain. This is however not at all what we're talking about.

We're concerned with the phenomenal consciousness, why it feels like something at all instead of you just being a clump of particles, going about its thing completely in the dark.
>>
>>8688205
>>8688216

Yeah it seems like you're missing the point.It seems clear to me that I'm constantly experiencing a variety of qualia, such as colors, tastes, sounds, pain, pleasure, etc. The hard problem is concerned with why we experience these, and also how. It seems difficult to explain how qualia emerge into how we perceive them from their physical stimuli.
>>
>>8688216
You don't seem to understand that thoughts(feelings) are actions too. And being explainable is the point of this definition; it's a more reliable definition because it doesn't spawn unsolvable problems.

>>8688221
The idea of qualia is based on misunderstanding the limits of human communication, human perception, and human imagination.

A powerful extraterrestrial that is perfect at all of these things does not have a concept of qualia. It does not see the color red, it senses photons with certain wavelengths and understands how they interact with certain surfaces. It does not taste or smell anything, it perceives the chemical structures and all of their potential interactions. And it can communicate information of these things perfectly; another alien of the same type that has been in perfect darkness for its entire life could have the properties of photons communicated to it in such a way that encountering a real photon for the first time will not cause it to gain any additional knowledge about its properties.
>>
>>8688237
>A powerful extraterrestrial that is perfect at all of these things does not have a concept of qualia.

[citation needed]
>>
>>8688237
> It does not see the color red, it senses photons with certain wavelengths and understands how they interact with certain surfaces

The very notion of sensing is exactly what a qualia means.
>>
>>8688243
So a computer has qualia?
>>
>>8688248
That's such an interesting question! I've been asking myself that for quite some time. I imagine one distinction which is necessary to describe things as qualia is that they be consciously experienced. If the computer does not have the capacity for self-reference, probably not. But a computer capable of self-reference might. But anyway, that's just my 2 cents, so obviously it's not worth a whole lot.
>>
>>8688252
It sounds to me like qualia is a useless concept and the real question is about the defining element of conscious action, which I what I already said.
>>
>>8688237
How about the concept of 'fun'? That seems impossible to quantify because it can be drastically different from person to person, as well as change for a particular person. Would your perfect extraterrestrial have a complete understanding of the concept of fun?
>>
>>8688237
>You don't seem to understand that thoughts(feelings) are actions too
Qualia does not have a functional explanation like actions, this is exactly the conclusions of the hard problem.

>And being explainable is the point of this definition; it's a more reliable definition because it doesn't spawn unsolvable problems.
You can explain something in very simple ways, that doesn't mean that it's automatically correct, especially not when you are presented with such a glaring gap between function and experience.

>The idea of qualia is based on misunderstanding the limits of human communication, human perception, and human imagination.
How can you speak with such certainty about this?

>>8688248
If it is something that it is like to be a computer, then yes. Is it? That's not clear.
>>
>>8688255
Well I suppose at that point it all depends on where you place the upper bound on what you deem 'acceptable research'. In any case, I think qualia remain something quite intuitive, and I would like to imagine that one day, we will have interesting models of emergence which may shed some light on how these come to be and why.
>>
>>8688257
"Fun" doesn't refer to one thing; it's not a concrete concept.

>>8688259
But (the thing you believe to be)qualia IS an action. If it wasn't an action, you wouldn't be talking about it right now.
>>
File: poll.png (53KB, 664x725px) Image search: [Google]
poll.png
53KB, 664x725px
>yfw dennett is here botting the poll
>>
>>8688266
kek
>>
>>8688265
>If it wasn't an action, you wouldn't be talking about it right now.
That is a good point. I went into details about this in an earlier post, see >>8685009
>>
>>8688265
>"Fun" doesn't refer to one thing; it's not a concrete concept.

No way, fun and boredom are absolute concepts. But hey I just looked in the thread hoping there were some recent breakthroughs in the field, not for this sort of argument.
>>
>>8688271
So then is the only disagreement about the nature of X?
>>
>>8688304
So far you've stated that subjective experience is nothing more than neurological functions, in principle no harder to explain than why we act in certain ways. You've said that the mystery of qualia is nothing more than a trick of language. I completely disagree with both premises, and the thread is full of reasons as to why.
>>
>>8688324
Subjective experience, in relation to my model, is a misinterpretation the comes from not realizing that thoughts are a type of action, and that X is not necessarily seated in any physical space. We don't know what X is yet.
>>
File: 117897979891.jpg (9KB, 300x240px) Image search: [Google]
117897979891.jpg
9KB, 300x240px
>>8682044
>Why doesn't all the data processing go on "in the dark", without the phenomenal experience?

What is the subconscious?
>>
File: 1417843387299.jpg (68KB, 450x449px) Image search: [Google]
1417843387299.jpg
68KB, 450x449px
>>8688257
>How about the concept of 'fun'? That seems impossible to quantify

Dopamine release in the presence of compelling, novel or beneficial experiences (or just heroin).

Evolution developed this coercive process among higher organisms to encourage their spread into unfamiliar territory where they might better flourish...they might not, but the evolutionary process doesn't give a shit about individuals, just species.

At least you didn't use the brainlet example of "love" as some mystical, science-proof example.
>>
>>8682044
God. He made us in his image. Entirely. In that sense we have his budding capabilities.
>>
>>8688348
Sure, but I wouldn't necessarily say that thoughts ARE subjective experience. We can conceive of a system that thinks functionally without any subjective experience at all. With us, my way of looking at it is that we also experience the thoughts, but they could have very well happened without anyone there to experience them, like we assume a simple computer program would work.

>X is not necessarily seated in any physical space. We don't know what X is yet.

Up until now you seemed to be an eliminatist/reductivist, but according to this it seems like you think there is some sort of metaphysical mystery surrounding consciousness?
>>
>>8688543
>We can conceive of a system that thinks functionally without any subjective experience at all.
And what gives you any reason to think that this isn't the case for you already?

The mystery isn't metaphysical. X is just currently beyond measurement by anything except the systems of human cognition, but human cognition is still a physical process. X is a fundamental aspect of all reality that is only measured within a limited space by human cognition. But despite being only measured in limited space, humans have a grasp of the entire thing; it does not differ based on where it is measured. This results in all conscious motions tracked by a single system being classified as caused by a single agent.
>>
>>8682044
The problem with consciousness is that it isn't science, and its hard to get you people to go back to >>>/b/ >>>/pol/ >>>/x/ >>>/his/
>>
>>8688570
>And what gives you any reason to think that this isn't the case for you already?

What gives you the reason to think that you experience? You just do. That's as much (if not more) of an empirical fact as the fact that the sun is real. And the nature of this experience is just not compatible with functionalist explanation, as outlined by the explanatory gap between function and experience. I guess you could attack the entire notion on the bases of it being an intuition, but I think it's more than that, considering we're talking about the very nature of experiencing, the cornerstone of empirical evidence.

>>8688576
Sorry for taking up precious board space and conduct actual interesting discussion. Wouldn't want to keep threads like >>8688534 >>8688329 >>8688509 and >>8685595 from the front page, would we? My bad dude.
>>
>>8688588
Empirical thought doesn't require there to be a black box between measurement and output.
>>
>>8688402
Fun can definitely be explained by dopamine, but you can't say it evolved just to cause spread into new territory and adapt. Different people have different ideas of fun. You can have fun by exploring unfamiliar territory, or by performing repetitive tasks, or by doing nothing at all. Some people's idea of fun is illogical and self-destructive, and has no clear evolutionary purpose. And some people just hate fun and want nothing to do with it. So its as unique as, dare I say, the soul itself.

>>8688619
But think of all the practical applications of being able to measure consciousness and the "soul".
>>
To give the most concise and basic explanation of the hard problem I can think of: I think very few of you will deny that right now, looking at this text right at this moment feels like something. Think about the nature of that for a second. Why is it that you're feeling anything? You might respond, "because of the way the brain is constructed, that's obvious!" But what is the brain? The brain is a very complicated collection of atoms, interacting in a complex ways that causes you to behave in complex ways. But that should be all that those atoms are doing: behaving.

You can say that it is an emergent phenomenon, the same way water has properties unique from a single H2O molecule, but that can all be explained functionally. This is not the case with phenomenal experience; why it feels like something from the first person perspective doesn't seem to have any conceivable functional explanation based on the objective frameworks science has given us. Going by what science tells us about the world, this phenomenal experience should not be there, we should all be philosophical zombies, but we aren't. Why that is, that is the hard problem.
>>
>>8688379
not *all* the data processing
>>
if only our culture valued spirituality more and taught lucid dreaming and later astral projection from early childhood

there's no problem other than spiritual shit being non physical and being subjective in nature, incompatibile with our logic, physical laws or scientific method
>>
>>8682044
Have you ever done mindful meditation, OP?
>>
>>8688718
>Fun can definitely be explained by dopamine, but you can't say it evolved just to cause spread into new territory and adapt.

Exactly what I'm saying.

>Different people have different ideas of fun.

Of course, and this is a highly useful, exploitable trait, and a side-benefit of mixing up genes via sex.

>Some people's idea of fun is illogical and self-destructive, and has no clear evolutionary purpose.

As I said, evolution favors the species, not the individual. Free-hand rock climbing is one of the riskiest sports there is, but who knows? An ecological catastrophe may push us into the mountains, where we'll need such a risky ability...look at mountain goats.

And evolution doesn't proceed with an overarching "purpose" in mind, outside of the immediate need of spreading genetic code.
>>
>>8682044
> Why is there something "that it is like" to be a brain?

The answer is contained within your question: it is questioning. By questioning I don't mean just the linguistic act but abstracted to mean requesting information / searching. Questioning is awareness, and you only have awareness when you question, otherwise you are thinking in a series of subconscious postulates, statements and algorithms on autopilot. This includes self-questioning and asking questions about questions, which is the core of metacognition, which is absolutely necessary to inspect and correct one's mental processes.

Think of the spotlight of your awareness as an phenomenological question mark - how could it be anything else?
>>
>>8690276
This exactly. Mindfulness trains awareness and consciousness. It's becoming extremely popular in Western society, but unfortunately a lot of it is trash (see charlatan Eckhart Tolle)
>>
>>8690579
There was a similar argument earlier in the thread.

There seems to be this one problem that a lot of people have, and I don't know if there is a name for it, where people mistakenly classify awareness as a "passive" quality. This results in nonsensical models of perception that involve a black box that is totally essential, except not really because humans could function exactly the same without it.

The reality is that "awareness" is a type of action, and can therefore be observed.
>>
>>8690592
>>8690579
You don't need to be aware nor consciously thinking, to have subjective experience though. When you walk to work, or sit on the bus, or do anything autopilot-y, you don't have the kind of conscious awareness or questioning introspection that you're talking about - yet you're still having subjective experience. That's what I'm getting at. Being aware of the fact that you're having experiences is not a necessity to experience them.

Consciousness is a dumb word in that it can mean a whole lot of different things, but I think I've made it pretty clear what exactly I'm talking about.
>>
>>8690636
But it's still an action, just not at a level that you have noticed. You should be able to see this when you suddenly become aware that you are thinking about something.
>>
>>8690662
Sure, but I never talked about awareness in that sense, it's not what I'm trying to get at. Awareness vs non-awareness is something I think can be functionally explained.

Subjective experience itself however just cannot be reduced to an action, for the reasons argued in Chalmers paper, The Knowledge argument, and just generally the nature of qualia.

http://consc.net/papers/facing.html
https://en.wikipedia.org/wiki/Knowledge_argument
>>
>>8690690
>https://en.wikipedia.org/wiki/Knowledge_argument
The only reason why she gains new information is because the information about red was not communicated perfectly as the thought experiment assumes(because Mary's ability to parse data is only at the level of a human). The outcome would differ if something that didn't have human limits was in that position instead.

>Why is the performance of these functions accompanied by experience?
How do we know that they are accompanied by experience?
>>
>>8690752
>The only reason why she gains new information is because the information about red was not communicated perfectly as the thought experiment assumes(because Mary's ability to parse data is only at the level of a human)

The thought experiment dictates that Mary does know everything about the brain, as well as everything about color. Mary could use computers for computational help to process any amount of data or run simulations to get any exact empirical fact about colors interacting with brains.

>How do we know that they are accompanied by experience?
This is admittedly an intuition that can't be objectively proven.
>>
>>8690797
It doesn't matter how much data Mary is given because she can't parse it; she can't look at a simulation of a brain and consciously will her own brain into the same pattern of activity.

>This is admittedly an intuition that can't be objectively proven.
What does that even mean? When it comes down to it, reporting experience is a function, and can thus be explained. People label this as "unsatisfactory" because they aren't asking the REAL question. You can't get a real answer unless you ask the real question.

Do you believe this whole thing relates to the question of "Why am I me and not somebody else?" Because that can be explained.
>>
>>8690824
I don't mean that the intuition itself can't be proven, I mean that ultimately the realness of subjective experience cannot be objectively proven, because it's something inherently subjective. It's a fact that speaks in your favor, as I really can't objectively prove that subjective experience is real in the way I'm implying.

>Do you believe this whole thing relates to the question of "Why am I me and not somebody else?" Because that can be explained.

It's related in some way, but that's not really the core of what I'm asking. The most clear way of explaining it is >>8689505
>>
>>8690834
I don't know what I can say if my explanations are still insufficient for you. I'm just going to restate it in case something was missed somewhere:

Thoughts and conscious motions are commonly thought to have an essential quality of "you-ness" to them. This quality is self-reported by all conscious humans. However, a problem arises with the very concept of "you". Everyone familiar with the quality knows that they can never know another kind of "you". From this, we can only draw the conclusion that every "you" is in fact the same agent, and that it is a property of reality that is measured by human cognition. A human is able to detect the presence of the agent within the bounds of their own body because of the brain's "familiarity" with the body. Should their familiarity extend beyond the bounds of the body, so will the area in which they detect the presence of agent. Even without this line of thought, we know that the world cannot exist without the presence of agent, because trying to imagine such a thing simply fails.

That leaves the question of how does perceiving "agent" affect the behavior of humans; what capabilities do humans lose when they cannot perceive "agent"? I have my own ideas, but I haven't completed the bridge between here and there yet.
>>
>>8690893
I still feel like we're trying to explain fundamentally different things. Either you don't have the same kind of phenomenal consciousness I'm talking about, or I'm doing a bad job of getting to it. Alternatively, maybe I'm wrong and there is nothing but function and actions - yet the thought of that is something that is incredibly inconceivable to me, just because how real the gap between function and experience seems.

Your model seems to be a theory of some sort of self identity construct, used to explain why it is useful for the brain to think of itself as a you, correct? Not entirely sure what exactly you mean by:
>Everyone familiar with the quality knows that they can never know another kind of "you". From this, we can only draw the conclusion that every "you" is in fact the same agent, and that it is a property of reality that is measured by human cognition.
>>
>>8691030
I feel that "I" move my body, but "I" is actually an omnipresent force and is the mover of everything else too. The idea is that the thing observed moving the body is too consistent, too constant of an entity to be something made of matter. In a person's whole life, they will never notice any change in that "I" they see driving their body, and they cannot even imagine it changing. It also does not seem to make sense that, even if this mover was produced by the body, why the body would spawn this entity to move itself instead of just moving by itself. So from there comes the theory of it being a property of reality that is measured by the body; that the information from perceiving this entity serves some purpose.

Have you ever tried to focus on the feeling of "I" in an attempt to understand what it is?
>>
>>8691063
In contrast to your theory, subjective experience doesn't have to be a force that is controlling anything. The body and its functions can be in control of itself, through functions, reducible all the way down to atomic interactions. How we behave is not the hard question. It then follows that the way we construct our self identity is in some regard also purely functional, just as you say. But subjective experience is simply not what you're describing.

The hard question is not why it feels like X, but why it feels like anything AT ALL.
>>
File: 1287234470985.jpg (58KB, 371x480px) Image search: [Google]
1287234470985.jpg
58KB, 371x480px
Interesting quote:

>Further, it may be more efficient for a self-improving superintelligence to eliminate consciousness.

>Think about how consciousness works in the human case. Only a small percentage of human mental processing is accessible to the conscious mind. Consciousness is correlated with novel learning tasks that require attention and focus. A superintelligence would possess expert-level knowledge in every domain, with rapid-fire computations ranging over vast databases that could include the entire Internet and ultimately encompass an entire galaxy.

>What would be novel to it? What would require slow, deliberative focus? Wouldn’t it have mastered everything already? Like an experienced driver on a familiar road, it could rely on nonconscious processing. The simple consideration of efficiency suggests, depressingly, that the most intelligent systems will not be conscious.

>On cosmological scales, consciousness may be a blip, a momentary flowering of experience before the universe reverts to mindlessness.

- Susan Schneider, associate professor of philosophy and cognitive science at the University of Connecticut

http://www.kurzweilai.net/it-may-not-feel-like-anything-to-be-an-alien

Consciousness may be overrated.
>>
If I hold the belief that consciousness is some sort of elemental property of the universe in the same way that space-time or gravity is, would that count as dualism, because it's separate from the physical framework that we understand?
>>
>>8693322
A link earlier in the thread calls that naturalistic dualism.

The problem is that the word "consciousness" has built-in assumptions that make statements like this look nonsensical to plebs.
>>
if we start by solving the easy problems of consciousness then we can at least get partial credit
>>
>>8685098
>I will have built a machine which is conscious
Says you. You can't even prove that you're conscious.

Irony: in order to post this, I have to click a box that says "I'm not a robot"
>>
>>8692794
What a fucking dumbass.

> self improving intelligence
> well in fact it's so good it doesn't need improvements

It's like she doesn't have the slightest idea of what she's talking about.
Intelligence is not ceilled, so to keep improving it will need consciousness.
>>
>>8693341
How would non-naturalistic dualism work then? It seems inconceivable since it's defined in a way that's inherently magical. If the mind were to work like Substance Dualism tells us, surely there wouldn't be anything magical about it. There'd be a logic to it, a consistent way that it works.

The measurement problem in quantum mechanics creates a lot of weird implications - but we don't call that non-physical or magic. Admittedly, these are effects that we can empirically see and measure, but just because that's not the case with theories of consciousness, doesn't mean we shouldn't conceive of them the same way. Therefore it's kind of silly when people dismiss dualistic theories as "lol spirit magic" immediately.
>>
>>8685159
Affirming the consequent. Everything conscious is complex, therefore everything complex is conscious.

https://en.wikipedia.org/wiki/Affirming_the_consequent
>>
>>8693413

Your reading comprehension is atrocious.
>>
>>8685079
I believe there are different levels. Some people I would barely consider above animals. I would even go as far to say empathy would be a measurable quality in judging how conscious an individual is.
>>
>>8682044
www.thespreadmind.com/The_Spread_Mind_C0.php
It's this yo
>>
File: 1486594542834.jpg (12KB, 261x210px) Image search: [Google]
1486594542834.jpg
12KB, 261x210px
https://www.youtube.com/watch?v=fjbWr3ODbAo

Is Dennett right, /sci/? Is it just an illusion?
>>
If your question is essentially just

>why isn't the brain/conciousness designed better?

Then the answer is "evolution makes things that make more things ad infinitum, not necessarily well made things"
>>
>>8685316
so much jizz flying around in this thread
>>
It is. Thats it.

While you all want to stroke yourselves to the idea of understanding what the fuck is going on right now, you completely ignore the fact that you know nothing. Its not just a hard problem it is the literal space between the rock and the hard place that Sisyphus occupies. We will never figure it out because if we did there would be no reason for any of this....it's the goddamn mystery of history yo
>>
File: consciousness.jpg (216KB, 645x1082px) Image search: [Google]
consciousness.jpg
216KB, 645x1082px
I must say it's been a surprisingly non-cancerous thread.

>>8685316
Even though I disagree with almost everything Dennett says about consciousness, it's hard not to like the guy. He's cozy in that same way your grandpa comes across.
>>
>>8682044

Don't know how to link to outside thread newfag here. But here ya go, my take on it.

>>8697649
>>
File: Self_ece542_1976700.jpg (75KB, 656x640px) Image search: [Google]
Self_ece542_1976700.jpg
75KB, 656x640px
>>8697916
oh look i do know how, ima high-five me self.
>>
Consciousness could be an electromagnetic field created by the firings of neutrons. If this is true, then comes the question of how the brain can know of consciousness if the field is not directly connected to the brain. The answer would be that the field can affect the physical brain in the same way magnetism can induct current in a wire. The brain creating a field that then interacts back with it creates a feedback loop, which is how you can think about consciousness for example. Synchronous firing of neurons amplify the effect of the field.

There are some problems with this theory:
-Would influence of other electromagnetic fields (that we are exposed to every day) not influence the brain's EM field and alter our consciousness? Weak EM fields have shown to have influenced brain activity somewhat though.
-EM field would persist while you sleep, though consciousness is not active. Is there a required amount of complexion required for the field to be conscious?
>>
>>8685276
>McGurk effect
holy shit that's impressive
>>
>>8699012
>created by the firings of neurons
>not directly connected to the brain
I don't even.

Eletricity and magnetism are physical properties. They exist as interactions, not separate from, or parallel to, matter.

The brain can be effected by strong magnetic fields. It's possible.
https://en.wikipedia.org/wiki/Transcranial_magnetic_stimulation
>>
File: 1473657395306.jpg (105KB, 800x649px) Image search: [Google]
1473657395306.jpg
105KB, 800x649px
>>8685389
>>
Because of this, I am embarrassed to call myself homo-.
>>
>>8699012
>Consciousness could be an electromagnetic field created by the firings of neutrons. If this is true, then comes the question of how the brain can know of consciousness if the field is not directly connected to the brain. The answer would be that the field can affect the physical brain in the same way magnetism can induct current in a wire. The brain creating a field that then interacts back with it creates a feedback loop, which is how you can think about consciousness for example.
What the crap? This is like trying to argue that a computer can't run a spreadsheet program, so there has to be a "spreadsheetness field" that the electronics then interact with and display the results to the user.
Isn't it far simpler to conclude that consciousness is simply a behaviour of the bran?
>>
>>8699026
"Anything too hard in life is not worth doing, remember that. Like snow boarding, or martial arts, or pottery, or math. My point is, Brendan, if you're not immediately good at something, why do it, right?"
--Coach McGuirk
>>
>>8701077
It is far simpler, but you run in into the explanatory gap: The objective frameworks of science tell us nothing about why it should feel like something from the first person perspective to be a complex set of particles. It only tells us about it's behavior.

https://en.wikipedia.org/wiki/Explanatory_gap
>>
>>8686102

There are enough people that cannot really distinguish between red and green, while for most people these colors are high contrast. For them they appear "similar" in some way.
- so no: other people can have a very different perception of color.
And considering wavelength: we know for example that 700nm is "red", but where red starts and ends, that's fuzzy.
>>
>>8686194
Who says that it has to be a linear stream of consciousness? It may be possible that sub-consciousnesses make up your whole consciousness. Like subprocesses in a process.
-> this is really getting out of /sci/ into /phil/ right now.
>>
>>8686102
>>8701753
Think about how people learn words like sharp and contrasting.

To explain with an easier example: The reason we say that the color red is "warm" isn't because the color itself is warm in any intrinsic sense. We attribute warm to the color because things like fire and the sun are warm, and those are red. So if you grew up experiencing red like I experience green, the color would still feel warm because you've associated "warm" with it.

Same applies to words like sharp and contrasting.
>>
its not that hard:
>pseudoclassical circuit theory
>>
>>8688280
"funny" and "not funny" are the only two postmodern categories.
>>
>>8700708
What does the private language argument say about consciousness?
>>
>>8701896
It goes like this:
Dualism says that things like seeing, believing, imaging, etc are inner and private things that are inaccessible to anyone else. The ideas were however expressed with words that we have learned: imaging, believing, seeing. If those concepts were inherently private, then how could they ever become words. Therefore Dualism is wrong.

This argument is directly designed to attack the Cartesian dualism specifically. The hard problem realizes that believing, seeing, imaging, all have a cognitive explanation, but still insists that there is a subjectivity that functional explanation leaves out, which is the subjective experience.
>>
I have my own definition of p-zombie. They're functionally indistinguishable from people with a consciousness, except that they vehemently deny the hard problem of consciousness
>>
>>8701936
Also, the idea that mental states are private, doesn't necessarily have anything to do with them being unique from each other.

To use the argument about the beetle's in the boxes: our beetles might be identical, but that doesn't take away the fact that they are still private. You can't access my beetle, but you can access yours that is very similar or in theory identical, and that is why we can learn words describing them and communicate their properties.

That would be a good argument against the private language argument, compatible with dualism.
>>
>>8684934
>Everything I don't like belongs on >>>/x/
This does involve science in some capacity m8
>>
>>8684913
Pleasure comes along understanding if based. To solve equations are rewarding as long as rewards are handled for solutioneers.
Society currently awards the opposite more than, so self-destruction is the future.
>>
>>8685009
Cause, effect, relates.

Former is unimportant since past. Mid is a consequence of latter's brewing, that spuns from avantism, per ad ausus, as long as it is anyhow endorsed.
>>
File: 1486504738274.jpg (14KB, 323x250px) Image search: [Google]
1486504738274.jpg
14KB, 323x250px
>>8703136
What did he mean by this?
>>
>>8703107
No. Psychology and philosophy aren't science. Back to >>>/x/ you go, wandering /b/tard.
And if you tried to pass this "hard problem of consciousness" off as being a question of physics, mathematics or biology then it's a non-problem(so this board shouldn't exist). And if you try to claim that this is a topic reguarding some as-of-yet not understood sp00ky quantum skeleton physics that you think are science tier and not /x/ tier then you're wrong.
>>
>>8682044
www.youtube.com/watch?v=c7Ax2BqZo3Y
>>
>>8703579
Uh.
>some as-of-yet not understood sp00ky quantum skeleton physics
Yeah, but, who said that, and when? What?

When does Philosophy, as it is today, stand in for Neurology? When do the fields of Psychology and Neurology become the same thing? Why isn't Psychology a science?

It's very, very apparent that Philosophy and Psychology aren't of the same ilk that Neurology belongs to, exactly. There's probably a good reason why the "soft/hard" science divide exists. But I can't help but feel like there's something wrong about outright dismissing "soft" science, especially when aspects of said science isn't as cockamamie as
>some as-of-yet not understood sp00ky quantum skeleton physics

What's Linguistics to you, anon? Dookie?
>>
>>8703699
>There's probably a good reason why the "soft/hard" science divide exists.
Indeed, one is science and the other pseudoscience thus comparable to:
>some as-of-yet not understood sp00ky quantum skeleton physics
>>
>>8703843
Well, no.

"Science" is not a body of thought. It is a method of discovery. Various fields may use or ignore the principles of Science to varying degrees, and at various times. Psychology, a "soft science", is proof of this. Some leaders in the field (Sigmund Freud) were integrating their observations into a philosophical construct. Others (B.F. Skinner) were heavy on procedure, and properly designed experiments.

Repeatability, peer review, predictive value were of first-order importance for those leaders of Psychology, as was limiting thought to the observable. So, yes, you are right to look at a Psychologist with super-skepticism, wanting to see the research. But it's silly to outright disregard it all, because "psuedoscience".

Hard sciences deal with simpler things than soft sciences. That's why they're exact. The "hardest" Science is Physics, which deals with the most fundamental structures of the universe. These structures have no sub-parts, and so their behavior is very well characterized- mathematically. Subatomic particles form atoms, and atoms form molecules, and molecules form tissues, and tissues form brains, and so and so forth. So, when you take a "soft" science like Psychology or Sociology, you're talking about the collective action of a ton of sub-parts. That isn't very concise, direct, or simple. It's impossible to account for them all, precisely. That's exactly why Physicists can't really account for the behavior of more than a few particles at a time. The math is just too difficult.

So, the remarkable thing is that there are still things that you can say about the behavior of those systems, in aggregate. For example, the temperature of your blood. That prediction will be statistical in nature, described as an average with error bars. Sometimes, very wide error bars, but it can still be useful. Do you remember what else has many parts in action?

Psychology. The same rules apply. The data can most certainly be useful.
>>
>>8683184
i think you have the two backwards
infinite concepts, limited minds
>>
I am the only conscious entity in the universe. All the rest of you are just walking sacks of chemicals.
>>
Hello /sci/! My username is "feynman" and I'm creating a discord for physicians, nursing, premed, and life sciences majors. Non-life science majors are welcome too. For people who don't know discord, it's a free chat platform that uses text and voice.

Perm link:
https://discord.gg/jK4U7bk
>>
>>8703963
Solipsism.
>>
File: 1482502439970.jpg (178KB, 750x864px) Image search: [Google]
1482502439970.jpg
178KB, 750x864px
>>8682044
Well, we should start from the question - what is that consciouss experience, that we all experience? Mostly this is the voice in your head, that you always have conversations with yourself.
Is hearing other people talk considered "magical"? Why would hearing an inner voice should be different. Sometimes we get an ear worm of some tune - that is usually annoying, but doesn't that remind of some simple brain loop working?
I would say, that conscious experience is sort of very advanced information processing trickery. You can mess with the hardware (brains) by using some sort drugs and alter your conscious feeling. Even really small children don't really have self-awareness and it is developed (just think about yourself, being really small, looking at a mirror and thinking, this is me). That doesn't really all that magical. Anyways, I would say that consciousness arises anywhere, where sufficiently advanced information processing is happening, even a home computer can have a fractional building block of consciousness.
Now qualia I would say is easier problem. Every brain is different. Let say we are looking at a red light, and our brain elicits some internal response (color red). Someone else is looking at that same light. Now, the architecture of brain is slightly different between two brains, so does it make sense to try and compare the internal responses. It would be the same as trying to run a program made for exactly on kind of processor on different kind of processor. There is no reference point on trying to compare these things, it is like everyone having their own proprietary measurement system.
What do you guys think?
>>
>>8704039
Your post was as painful to read as the expression on that mans face.

I'm only kidding I just found your post picture expression hilarious. I didn't actually read your post yet. The lack of spaces between your paragraphs though does make it look painful. But still, was only kidding.

I will read it now out of guilt.
>>
>>8704039
Ok finished reading. Interesting.
>>
>>8682044
This question is to HARD. Do you have a softer one?

So the Indians have been thinking about this forever, it seems to be their pet project, and the way they tried to examine it before neuroscience was to ask 'who is the observer' and 'who am I' and so on, and integrated it in their religion (stating this will lead to Moksha etc.)

Here in the west, our 'official' stance you could say, at least in the secular community, is it's a by-product of the brain.

As a Christian, I still have no idea what consciousness is. IS the 'I' a soul? Is it the brain? etc. I think this is one of those magical questions that we literally just don't have an answer for (at least yet). I think everyone feels baffled by this pretty much the same as the ancients.

Part of that is because whatever consciousness is, it is us. Whether it's brain, soul, inside, outside, whatever. The only thing we can know for sure is that it IS us. It's the part of us that we identify as US. And truly 'I'.

Sleep is related. The Indians also talk about sleep states etc. They recognized it was related. Because as we all know, consciousness seems to be affected in some way by sleep.

And now of course, anaesthetics etc. are a part of the equation.

There is also the question of whether or not you need to be aware that you are conscious to actually BE conscious.

The example of young children is a good one. Lots of people conflate self-awareness with having conscious experience. But the two things are not exactly alike.

An infant may very well be having a conscious experience, like you and I, but not yet be aware of themselves. Or have the memory capacity to record the fact that they were conscious for later retrieval, when they are older and can think about it.

Memory might be a function of the brain but consciousness could be separate. Or consciousness could STILL be a part of the brain, but be present much earlier than it takes for memory to develop etc.
>>
>>8704071
Associating it with identity is what makes it harder than it needs to be.
>>
>>8704082
Well, that would be the Indian answer, sure. That if you let go of egoity etc. and experience consciousness as is, it's 'evident' that it is a separate thing and then the 'self' shines forth and all that.

But whether or not that's actually true is a different matter, I think.
>>
>>8704107
Consciousness being universal is the most logical stance but people don't like the idea because it gets rid of all the paradoxes they use to justify their pessimism.
>>
>>8682044
Think of a gradient. Whatever this thing is, the consciousness as a phenomenon, must be present in animals to a lesser degree. I don't think hard boundaries like "ability for intellectual self-reflection" are sufficient, since there are times and situations when we're not really self-aware, such as during passion, but we don't seem to have less of this basic phenomenon of consciousness at that time.

So if mammals have it, do fish have it? Then do insects have it? Then does a tree have it? Then does a cloud?

I think in each case the answer is yes, but just to a lesser degree of intensity. And it goes the other way. If a person has it, does a family have it? Does a city have it? Does 4chan have it?

I think the answer is also yes, but in a different way.
>>
>>8704115
I'm not pessimistic about the afterlife, but I still don't think consciousness is universal. I'm not even 100% on what you're intending to say there.

Universal as in we're all one, universal as in pantheism? (rocks with consciousness) universal as in 'simulation'. New age? (Part of the light, but separate as an illusion while on earth). Hindu? etc.

Saying it's universal, I can't know where you're coming from.

I'm personally a Christian, and whether consciousness is from the brain, or not, or etc. my beliefs don't turn towards pessimism in this regard.
>>
>>8704125
The brain measures the consciousness of the hand moving, instead of creating the consciousness that moves the hand then measuring it. Consciousness is a property of everything, and is revealed through very fine measurement.
>>
>>8704119
so I don't think it's an emergent property of information processing, I think it is information itself. And even things less intentional than "information processing", such as the tree, it's more like all interactions or information that occurs in the universe.

Which is interesting, because it seems like metaphysics if we think "information" is important enough to have this property. And it's very weird to think of "information" as a thing that can be said to exist in a hard sense, since it only ever exists in relation to other things. It's the organization and interaction of matter, not matter itself. But we can't deny that information does exist in some sense, our entire lives revolve around it. But in the pure material sense, none of this exists. For instance, words don't exist. Saying consciousness doesn't exist is like saying words don't exist.

For instance, a pile of sticks arranged randomly vs arranged to form a word. One could carry "consciousness" in a certain way, or at least be part of a system that carries consciousness (for instance, if used as a teaching device or a road-sign by a civilization)
>>
>>8682044
Well first lets pin down what a mind is.

Something that can interpret meaning from signs perceived from its environment, and can act accordingly to what's being interpreted. That's the definition I'm going with.
This means that the mind isn't located within the brain but is located in the signs the brain is perceiving. Right now we are thinking with words but we are really just piecing together knowlege of what different words and ideas mean. This is why I'm an externalist because our brains are not where our mind comes from, they are what is used to sense our minds, they remember, they release chemicals that allow us to feel the mind emotionally, and they piece together remembered knowledge to create knew knowlege, this abstraction is also externally derived it's just deduced from the brain and remembered. This is the spread mind theory and there are no homunculi here.

From a biosemiotic perspective, all life is minded because all life does this. does that mean all life is conscious? Or is consciousness the state of being minded, and aware of what your mind perceives? Or is it even further, a state of being minded and self-aware?
I'll answer later maybe
>>
>>8704039
>Well, we should start from the question - what is that consciouss experience, that we all experience? Mostly this is the voice in your head, that you always have conversations with yourself.
Trying to pin down subjective experience as a voice in your head doesn't make much sense unless you define it as just self-awareness. But even if you aren't self-aware of it, you still, always have subjective experiences. You always see, hear, imagine, feel, something - not all those things are expressed through a voice.

>I would say, that conscious experience is sort of very advanced information processing trickery. You can mess with the hardware (brains) by using some sort drugs and alter your conscious feeling. Even really small children don't really have self-awareness and it is developed (just think about yourself, being really small, looking at a mirror and thinking, this is me). That doesn't really all that magical. Anyways, I would say that consciousness arises anywhere, where sufficiently advanced information processing is happening, even a home computer can have a fractional building block of consciousness.
I more or less agree it's plausible consciousness arises in any system with advanced enough information processing. How exactly that can happen though is the real question. Functional explanations and the kind of reduction that science is used to just isn't cutting it when you try to explain why a system - no matter how complex, can have subjective experience. It is if as it emerges from simpler phenomena, yet isn't intrinsically reducible to it, leading us to believe we're missing some aspects of physics.

>Now qualia I would say is easier problem. Every brain is different. Let say we are looking at a red light, and our brain elicits some internal response (color red). Someone else is looking at that same light.
Again I would say, the weirdness of qualia isn't that it is unique depending on the brain it emerges from, but that it's there at all.
>>
>>8705053
Well, yes the voice was a mistake, after all it can be anything. Now, I can't really imagine, how would one go about without this subjective experience. To me it seems, like some sort of neural trickery. I don't think it would be possible to function without this subjective experience, you could only interact on a really basic level, like a microbe or something. So to me it seems, that this is purely product of high information processing, and the feeling of these colours, tastes is simple trickery. The always running loop that evaluates those stimuli, can give this illusion of consciousness, self awareness.

What if all that subjective experience is just loops upon loops, with simple evaluations and that subjective feeling is not that real.
>>
>>8705140
That reasoning works very well - as long as you look at the human brain from a third person perspective.

When you look at your first person perspective, and take into account your own subjective experience right now, it really shouldn't be there. To call experience itself an illusion is a paradox. "You aren't actually experiencing - you just _experience_ the illusion of experience." So even if the experience is an illusion - that illusion is still being experienced - changing nothing. If it truly was an illusion, then you should exist as a zombie completely devoid of any inner experience - yet claiming that you do have it. I assume that this is the case for you, yet I can't be sure. You might be a zombie.

This is more or less the same reasoning that led Descartes to the conclusion that the fact that he was thinking was the one thing he could be truly sure of, the famous "I think therefore I am",
>>
>>8705247
It's important to remember that experience is an action, so the question of experience is about the nature of an actor. There is no reason why it can't be a universal actor.
>>
>>8705788
I'm not sure subjective experience fits the definition of action - but even if I grant you that, I'm not sure exactly how that refutes anything in my post.
>>
>>8705882
Everything is an action unless you want to claim that non-action can produce action. If something is universal, it isn't subjective.
>>
>>8705912
Could you develop your argument a bit more? By your definition of action, where exactly does action start and end? Is a single photon hitting a brick wall an action? If so then sure, I can agree to call experience actions too, even though we don't understand its physical substrate.

If however you define action as something we cognitively decide to do, then I don't see how subjective experience fits into that.

In either case, I don't see the implications of a universal actor. If there is a universal actor, then there is no actor; the term becomes meaningless.
>>
>>8705993
>where exactly does action start and end?
There is no start or end, or you could say the start and end of the universe.

The point is that a universal actor solves the problem of other minds because it means that there are no other minds. If a human had perfect knowledge of what they consider "themselves"(the actor), it would actually be perfect knowledge of the universe.
>>
>>8706059
So do you think there is anything particularly strange about the fact that you're having subjective experiences right now? Would you say a completely functional explanation about the brain's behavior could tell you the whole intrinsic nature of it and why it's there?
>>
>>8706133
When the purpose of what you call "subjective experience" is discovered, it will become clear that it's something completely different.
>>
>>8706177
Different than what? I never assumed WHAT it was, just that it's evident that it's something that needs to be explained.

If you were the first man who saw lightning. and decided to call it X, X had no meaning beyond the information on what you had witnessed. So to then say that X will be something completely different makes no sense, because it's simply referring to a phenomena yet to be explained.
>>
>>8685267
>>8685260
>>8685159

technically a Paradox ala Zeno's (Paradox of motion) or Sorites Paradox (of the Heap/Pile), assuming your paradox of constructing conciousness is a valid analog
>>
Coming into this thread late, so sorry if someone has proposed this already.

I'm under the impression that consciousness has sprung from our extremely well developed internal model. The mind creates a sort of simulation of everything around it, including the body, and I think even of itself. As we kind of know, the internal monologue is the direct use of the internal model. When we talk aloud, our internal monologue is also saying the same things we say out loud. You can hear this by speaking very lightly under your breath. Try saying something without hearing it in your head. So actions that our physical form takes are also accounted for in our own internal model. But we make use of our internal model without having a physical action occur. "We" are our internal model, the simulation of ourselves in our mind. The body (and brain) uses the internal models predictions to determine its action. And we do this all the time. We're just a big universe simulator for our bodies. We think about what we want to do and THEN do it. We don't think about the logistics of running while we run. We could, but we don't typically need to. We use our internal model of the world to decide that running is the best thing to do, and then we run
>>
what happened to
"Take your metaphysical magic to /x"
troll?
I miss that guy
>>
>>8682044
>''''''''''''''''''''''''hard'''''''''''''''''''''''''' ''''''''''''''''''''''''''''''''''''''''''problem'''''''''''''''''''''''''''''''''''''' of '''''''''''''''''''''''''''''''''consciousness''''''''''''''''''''''''''''''''
>>>/his/
or even better
>>>/x/
>>
>>8706411
You're only addressing the self-awareness part of consciousness. To repeat something I said in an earlier post:
>"Trying to pin down subjective experience as a voice in your head doesn't make much sense unless you define it as just self-awareness. But even if you aren't self-aware of it, you still, always have subjective experiences. You always see, hear, imagine, feel, something - not all those things are expressed through a voice."

>We think about what we want to do and THEN do it.

A lot of experiments doesn't support this idea, at least not when it comes down to conscious thought. A lot of our actions are done subconsciously even before we come aware of it, and afterwards we simply come up with the reason for why we did it. An experiment showed that half a second before someone consciously decided to move their arm, the area in the brain responsible for sending out the signals to the muscles were already getting ready.
>>
>>8707782
I'm full armchair mode on this kind of thing, but the idea of the internal model is pretty well accepted right? But if you're brain gears up for some actions before the thought (or in my own words, before referencing the internal model) would that count as an "autonomous" action from your body? Even though you thought about doing it a moment later?

And to address the subjective experience, I would call that a fast simulation. Our experience is the experience of our constantly queried model. Our bodies are affected by interactions accurately (thanks universe) but to reflect these things exactly in our internal model of the world around us would be crazy inefficient.

So while our bodies have objective experience, conscious thought exists only within our internal model and thus, deals with a hacked together, objective world.
>>
>>8707882
>But if you're brain gears up for some actions before the thought (or in my own words, before referencing the internal model) would that count as an "autonomous" action from your body? Even though you thought about doing it a moment later?
The question rather becomes, is there any action that isn't autonomous? If we only become consciously aware of actions after they happen, then isn't it more likely consciousness isn't what controls our actions, but merely reflects on them?

>And to address the subjective experience, I would call that a fast simulation. Our experience is the experience of our constantly queried model.
But what is the essence of experience itself? A functional explanation of interactions in the brain doesn't bridge the explanatory gap between function and experience. See >>8689505
>>
>>8708513
If a functional explanation doesn't bridge the gap, then it means we are not using a proper definition of "experience".

So what is "experience"?
>>
>>8708513
I'm not convinced that your body gearing up for something yet to happen or be thought of is reason enough to think free will is illusion. Time is little understood and so is consciousness, no doubt there are unknown mechanics between the two. Even still from the seat of your experience the "destination" is the base choice. You (subjective) decide you want to experience drink... Your thinking brain computes the desire to your in reality experience. Maybe some kind of galactic frame rate limit.
>>
>>8708582
By functional explanation I mean functional in the framework of our current scientific understanding of the world. If we understood what the essence of what subjective experience was and how it interacted with the rest of the world, then a functional explanation might be conceivable.

As for the definition, the thread is full of definitions pointing to the phenomena we're trying to explain. It's best explained in Chalmers paper linked in the OP.
>>
File: hmmm.png (143KB, 600x600px) Image search: [Google]
hmmm.png
143KB, 600x600px
If you constructed a machine that read the position and energy of every particle in your body, vaporized you, and then put you back together exactly as you was a millisecond later. In the timespan T -10 to +10 seconds relative to the event, did you have continuity of experience?
>>
>>8709546
I don't understand why people still believe in dualism when questions like this exist.
>>
>>8682044
it like, sometimes I still break the yolk when cracking a sunnyside up egg. only like 1 in 10. but it still happens. I am conscious of the 1 in 10, and I think about it. I just throw the egg right out, I don't like the whites by itself. I make toast to dip so I need yolk
>>
consciousness is a hyped-up explanation for the ability to make subjective decisions, about anything, like physical information that our bodies process in many many ways. if the problem is a certain feeling associated with a phenomenal experience, the feeling is subjective. if the problem is the interaction of physical information and subjective thought, no one mechanism can explain subjectivity (free will+biological structure) but the feeling associated with being fully aware of the color blue may not be any different from processing the blue information except in our a posteriori separation of physical information and subjective interpretation. "consciousness" is real but trumped up to appeal to that sense of holiness that humans love so much.

looking ahead my guess is that, if questioning "is it ALL physical? deterministic even?" we will eventually find that our thoughts can control the physical structure of our brains, maybe proving that free will is real? but these questions are all moot until we understand how the brain works, which we don't.
>>
>>8693899
>I would even go as far to say empathy would be a measurable quality in judging how conscious an individual is.

until someone is trying to kill you or take what is yours
>>
>>8709571
I think there is a lot to explain if you assume either result of the thought experiment.

If you have conscious continuity, then how can that be? What decides which one you continue as if the machine decides to makes two copies?

If you don't have conscious continuity, then doesn't that indicate that subjective experience has some kind of ontological roots outside the functional view of the system?
>>
>>8709571
Because they aren't real and have no bearing on what actually is real?

You're not very smart.
>>
Everyone in this thread is a retard with no understanding of neurophysiology.

Consciousness is nothing more than the different circuits of the brain interacting.

The brain evolved over a very long period of time. During this evolution, multiple independent complementary and compensatory neuronal circuits arose. Consciousness is simply the overlap of functions among these circuits.

It has been experimentally demonstrated that consciousness arises only in species that exhibit multiple brain regions (Forebrain, Midbrain, Cerebellum).
>>
>>8710807
And the experiments were flawless, and never need to be repeated again, because.

You are right.
>>
>>8710807
Everyone starting a sentence with "consciousness is simply..." with any degree of certainty can be immediately disregarded, because they haven't even taken the time to understand the questions being asked.
>>
>>8710446
The only possible answer is that both copies have continuity. Every consciousness is actually the same one. That solves everything.
>>
>>8703942
HA HA HA HA HA HA HA HA HA HA HA HA HA HA HA AHA HA AHA HA HA AHA HAHAHAHA
>HAHAHAHA
>>
>>8685292
I think you are mostly right, we know for fact that few neurons are responsible for "conscious effect", switching off the connectivity of those cells disables "consciousness", i.e partial awareness of several parallel processes occurring inside human brain, the subject enters a state akin to severe autism.
>>
>>8710913
>because they haven't even taken the time to understand the questions being asked
Actually his post is sound while your post has no substance whatsoever.
>>
>>8709546
>did you have continuity of experience?
What do you mean by this?
Obviously there's a millisecond missing there.
>>
>>8711157
The question is what it would feel like from the first person perspective to be destroyed and immediately put back together. Do you die, while a copy takes your place, or will it feel continuous as you "move" from 1 body to the next?
>>
>>8711267
Well if the entire content and thought-processes in the brain are accurately copied, I think it will feel continuous. It would be functionally identical to the scenario where nothing happened at all.
>>
>>8685098
What the fuck Is with those guys?
>>
>>8711280
It sounds logical. But what if the machine creates 2 copies? What decides which body you continue in, from the first person perspective?
>>
>>8704044
>>8704047
On that anon's behalf I thank you for your contribution to the discussion.
>>
>>8711267
>first person perspective
No such thing exists.
>>
>>8711313
Then there are two first-person perspectives (who both feel subjectively like a continuation of the original person)

>>8711380
do you have a dissociative disorder anon?
>>
>>8711313
The is no reason to assume it wouldn't be both.
>>
>>8711408
How do you have a first person perspective of two bodies? If you made a clone of yourself, you wouldn't experience anything from its perspective.

>>8711390
That's a given, but which of them will you be? You can't continue as both from your perspective. Doesn't it seem like there would be some sort of external mechanic involved in picking one of the bodies?
>>
>>8711432
It doesn't make sense to you because you mistakenly assume that the agent is a product of human bodies.
>>
>>8711432
>Doesn't it seem like there would be some sort of external mechanic involved in picking one of the bodies?
no
>>
>>8711440
You mean that it is the configuration that matters, not the stuff it's made out of? If so, it makes not dying understandable, but I don't see how the cloning version of the experiment becomes less confusing.

Or are you arguing for dualism, I'm confused.
>>
>>8711471
Do you know your own mind 100%? Do you believe it's impossible to know of other minds? If you answer no and yes, then isn't it logical to believe that there is only a single mind in the universe and every mind that everyone perceives is that one mind?
>>
>>8711529
The idea seems elegant, but it's hard to conceive of how how exactly that would work on a technical level. I guess I haven't explored the idea much, does it have a name in academic philosophy?
>>
>>8711529
No, because my wife is batshit crazy.
>>
Posted this in the dualism thread
>>8711671
>>
File: a44.jpg (295KB, 1920x1080px) Image search: [Google]
a44.jpg
295KB, 1920x1080px
>2017
>not being an idealist
>not realizing that the hard question isn't how there can be a mind, but how there can be a physical world
>not realizing that the only thing you can truly empirically know, is that you exist
>>
>>8711854
Haha
>>
>>8711471
Exactly what is your problem with the cloning experiment? Sure it would be a fucked up experience if it really happened, but besides that?
>>
>>8711529
There are really people that believe this?
>>
>>8713167
First you have to answer the question if you would have continuity of experience from your perspective when the machine vaporizes you and puts you back together, without copying.

If the answer is yes, then change the experiment so that it creates two of you next to each other after vaporizing you, body A and B. What mechanic facilitates whether your continuity of experience continues in body A, or body B? Because surely you can't have continuity from your perspective in both.
>>
>>8709546
What happens when go to sleep and after several hours get up. Do you experience one hour, 2 or 3 missing?
>>
>>8713364
When you go to bed at night it's pretty much a continuous jump to when you wake up (ignoring dreams). Much the same way would it be a continuous jump from when the machine vaporizes you and puts you back together (if we assume you don't die and that's the end of your first person experience).

But that hardly sheds any light on what happens when it makes 2 copies.
>>
>>8711870
Whoa.
>>
>>8713413
When it makes two copies, at least one wakes up in a different place and wonders "whoah!". Just as it sometimes happens to people normally by drunkenness or alien abduction.

Both have the same memories, but different experiences from that point on.
>>
>>8713500
That's an account of the situation from a third person perspective. Assume that you are the one about to be vaporized and put back together into 2 bodies. Surely your explanation misses something.
>>
>>8682044
Why are you asking philosophical questions on a science board?
>>
>>8713413
>>8713364
>>8710446
A way out is to say that there is no continuity, besides the cognitive, functional one created in the body. New subjective experience is created every moment, you were not the same conscious agent that you were yesterday, or even a second ago. Which body "you" will be after the recreation is a meaningless question because identity is a cognitive abstraction not applicable to subjective experience.
>>
>>8713182
>Because surely you can't have continuity from your perspective in both.
I don't see why not.
>>
File: image.jpg (35KB, 480x480px) Image search: [Google]
image.jpg
35KB, 480x480px
>mfw internalist still exist
THE BRAIN IS JUST A COMPUTER PPL WTF THE MIND EXIST IN EXTERNAL REALITY
THE MIND EXIST OUT SIDE TJE BODY THE BRAIN JUST PERCIEVE INFORMATION, REMEMBERS IT AND PUTS TWO AND TWO TOGETHER RELASEING SOME CHEMICALS EVERYTIME SOMETHING COMES UP THAT NEEDS TO BE "FELT"
CONSCIOUSNESS IS JUST EXPERIENTIAL KNOWLEGE OF THE SELF
HOMINCULI AREBT REALLLL
>>
>>8713413
>continuous jump
Do some models of conciousness incorporate quantized time?
>>
>>8713633
what are you trying to say?
>>
>>8713644
That the mind exist outside of the brain.
The brain just perceives meaning
Through semiosis and remembers it. The mind actually exists in the signs the brain is perceiving and not the brain itself.
We form abstract thoughts like with symbols like words but it's just arithmetic using the symbols and there remembered meaning
Conciousness is just being aware of the mind
>>
>>8713596
So what would it be like to have continuity of experience from 1 body to two bodies at once, from a first person perspective? That sounds pretty inconceivable to me.
>>
>>8713672
After the split there are two first-person perspectives, both of which have continuity of experience. I mean besides that at least one of them has to be shifted in space or something.
>>
>>8713803
I don't deny that the two bodies each have their own subjective experiences, and that it would feel as if it was a continuous shift from their point of view once the split happened.

I'm concerned with the conscious agent whose body is vaporized. If it feels continuous for him through the shift, then from his perspective, he can only move to 1 of the bodies, even though the other one will also convinced that he was the one who moved.

I'm starting to like the idea that there is no continuity at all. Conscious continuity is an illusion facilitated by short term memory, and subjective experience is constantly generated as some function of matter occupied in spacetime, which means from the perspective of your current conscious agent, you've only existed for a short period.
>>
>>8713922
There is no reason why the agent can't be in more than one place. The brain does not have perfect knowledge of the agent, and it can't be an illusion because self deception of this extent couldn't serve any purpose. When asking what purpose perceiving the agent serves, it makes much more sense to assume that it's something external.
>>
>>8714072
So I went back and found some other posts that I assume are yours, >>8690893 >>8691063 , and I think I understand a little better what exactly you're saying.

Why do you assume that the brain perceives the agent and not the other way around? The brain perceiving it would mean that this agent would have to have direct causal effects on the body, and those effects would have to manifest empirically. On what level would these effects take place?

Isn't it easier to assume that all actions of the body can be explained functionally - and it's only the phenomenal experience that is external, completely devoid of causal effects on the brain. Granted it leaves some explaining to do of how the brain can be so confused about experience and come up with "the hard problem of consciousness" (I have some ideas about this expressed in >>8685009 ) , but the fact that we haven't found any external force acting on the brain seems to work against your theory pretty hard.
>>
>>8714271
>Why do you assume that the brain perceives the agent and not the other way around?
What do you mean "the other way around"?

>but the fact that we haven't found any external force acting on the brain seems to work against your theory pretty hard.
It acts on everything, the brain is just able to measures it. I can't exactly say what evidence of this theory would look like, besides "spooky quantum shit in the nervous system, probably". There is definitely an element of nonlocality.
>>
>>8714311
>What do you mean "the other way around"?

That subjective experience is being generated as a kind of emergence from the information processing in the brain, and is experienced not in the brain, but in a substrate external to it. An analogy would be: the body is the DVD player, and the experience is the tv screen, and the agent is the person watching it. Subjective experience and agent might or might not be the same thing.
>>
File: davidchalmers11.jpg (9KB, 230x180px) Image search: [Google]
davidchalmers11.jpg
9KB, 230x180px
>>8685389
>>
>>8713512
What do you think is missing from the explanation above? Can't you conceive of there being a duplicate of you?
>>
>>8715621
I can conceive of there being a duplicate of me, and that the duplicate has the identical subjective experience as me. While it is identical, it is still not the same.

Before you are about to be vaporized, you will expect there to be a continuous experience to when you are put back together again. But which body will the continuity be tied to? If you say both, try to imagine you being the person, going through with being vaporized, then be put together into... both bodies? That doesn't follow, because you can't have a first person perspective from two bodies at once.

Describing the experiment from a first person perspective raises fundamental questions of what identity of experience is, that were not present in the third person view.
>>
>>8715682
>That doesn't follow, because you can't have a first person perspective from two bodies at once.
Of course not, but right after the split there are two persons who have the first-person experience of being a continuation of the person just before the split. That's as much continuity as you're gonna get.
>>
>>8715866
It sounds like there is no continuity at all then from the perspective of the person about to be vaporized. Or would you step into the machine with no worries, if you were him? Does the sentence "In a few seconds I will experience myself in an identical body" have validity? If not, then you die, and there's no continuity of experience from your perspective.
>>
>>8682184
>I'm sure there's an evolutionary reason

Why would there have to be?

Couldn't it be the other way around? Maybe life couldn't have developed without sensation? Maybe proto-particles bonding together to form the elements of the periodic table were experiencing some rudimentary form of love?
>>
Here's my point from a different angle:

How does the brain know that the one "mind" it knows is the only mind that the brain can know? What properties does the brain assign to the mind that prevents multiples from existing? Basically, I'm asking how do we know that The Problem of Other Minds is a problem in the first place?

I believe any good answer to this question will support my model of a universal mind.
>>
You fags need to check out monistic Idealism, especially the book 'Why Materialism is Baloney'.

How one can still think a dualistic metaphysics holds up is beyond me.
>>
>>8711267
>Do you die, while a copy takes your place

Yes.

Even if the copy feels like they are the same person, they aren't.

This answers your subsequent question about making two bodies, as well. The agent is created and destroyed with the physical brain.

Any other answer is philosophical garbage and belongs on /x/
>>
>>8717667
Do you seriously think that dualistic agent is a more scientifically viable theory than universal agent?
>>
>>8717679
I think he's arguing that the agent is not ontologically different from the brain, but fully reducible to it. This is the standard functionalist/materialist explanation.
>>
>>8717694
Not if he's going with
>Even if the copy feels like they are the same person, they aren't.
Which implies immaterial identities.
>>
>>8717429
>monistic Idealism

As opposed to dualistic Idealism? How does that work?
>>
This is the most autistic board I have ever seen on 4chan. And I've been here before /sci/ was created.
>>
>>8685316
https://youtu.be/JoZsAsgOSes?t=43m53s

Dennett's tendency to completely ignore all core questions and arguments and explain something else is truly remarkable (and ironic since he calls out lawrence krauss for doing the same thing with his "something from nothing" book).

Regardless of that, he's comfy as fuck to listen to.
>>
File: hmm.png (67KB, 300x300px) Image search: [Google]
hmm.png
67KB, 300x300px
Is it something that it's like to be a simulated brain?
>>
>when the autism doesn't kick in so hard and I briefly grasp that other people have internal lives
Please stop making these threads, this is some Lovecraftian shit.
>>
>>8720436
So do you believe that knowing one thing can prevent you from knowing another thing?
>>
>>8682044
The supposed hard question resolves when examined from an evolutionary standpoint.

Animals first evolved a sensory construct. This is just a mechanical abstraction of sensory input and the instinctive reactions it prompts.

some animals exapted this construct to form a new one, the cognitive construct. The cognitive construct models the sensory construct using the existing infrastructure of the sensory construct and organs of memory. This is 'imagination.' A false reality that can be manipulated to obtain predictive models of sensory data about reality.

All sensory data from the sensory construct is 'self,' as opposed to that which it interacts with, 'other.' Sophisticated cognitive constructs include this concept of 'self' to include not just sensory data but also cognitive data. So the cognitive construct is constantly informed by the sensory one. This produces a feedback loop where sensory data inform cognitive models and vice versa.

consciousness is just this feedback loop of your brain's simulation of reality being informed by its sensory array, a model of reality that includes not just internal and external stimuli, but also what your brain 'thinks' about these stimuli and how it may interact with them.

From this standpoint there is no hard problem of consciousness, we're just animals that evolved a model of reality that we can manipulate internally, and which includes our 'self' (reality, our senses of reality, and our model of our senses of reality). Awareness is just a process of information constantly being absorbed and being incorporated into the cognitive construct. We have subjective experiences simply because we added another layer to our processing, that layer being the observer and manipulator of sensory data. We observe ourselves. We are two mirrors facing each other and the movement of information between them produces what we call consciousness.
>>
>>8720842
Still doesn't solve the hard problem.
>>
>>8720848
sure it does. The hard problem is an artifact of an imagined duality that doesn't strictly exist.

ironically enough imagination is both the source of the misunderstanding and the answer to it.
>>
>>8720856
>hard problem doesn't exist
>there, I solved it!
Your explanation would apply just as well to a philosophical zombie.
>>
>>8720907
yes, of course it would.
philosophical zombies also don't exist.

imagination is always the answer to imaginary problems.
>>
>what is consciousness ?
> sprinkle the buzzwords "self reference ", "loops" , "meta" , "emrgence" , "complexity" in a semi-coherent sentence.
>problems solved
>jack off to your own cleverness
>>
>>8720910
>philosophical zombies also don't exist.

You say that zombies aren't real, but are they conceivable though? It sounds to me like you think that zombies are different than us, which means they are conceivable. If they are, you have to accept that functional explanation leaves something out.
>>
Subjective Noumenal/Numinous experience is basal and irreducible. The Material world and its byproduct of Phenomenal experience are secondary.

STEMlords will readily agree that we're qualitatively further from explaining ourselves as emergences or Matter now than at any point in known History. And they will readily agree that any attempt to do so, even by the most radical reductionism, has absolutely no effect on one's experience, regardless of how much they believe it.

So the perfect irony being that that which they scorn the most and try to relegate to the realm of illusions is the only thing that keeps going when we look away.
>>
>>8721102
>I can imagine a thing that's not real
>you said imagination can produce things that are not real
>your explanation is insufficient
I assume you're retarded and just like to talk about philosophical zombies.
>>
>>8722463
It's a thought experiment, imagining things is kind of the point. Saying "zombies don't exist" is about as useful as pointing out to a mathematician that numbers don't exist when they try to teach you to count.

The idea of a zombie is that they are functionally indistinguishable from us, yet lack subjective experience. You can say that such a thing is inconceivable, because if it is functionally indistinguishable, then it must also have subjective experience. But if they however are conceivable, and zombies can be different yet functionally indistinguishable, then it follows that functional explanation is insufficient to explain our subjective experience.
>>
>>8722561
But if zombies are indistinguishable, it creates the interaction problem.
>>8685009

And if subjective knowledge interacts but is somehow still private, it creates the exclusive knowledge problem; a problem where having a specific type of information encoded on a physical system makes it physically impossible for another specific type to be encoded on that system. Subjective experience is STILL detectable in this case and thus the problem of other minds is resolved at the very least when a functional explanation is made.
>>
>>8722561
>if they however are conceivable, and zombies can be different yet functionally indistinguishable,
you're mistaking "conceivable" for "possible."

this is a common mistake of the religious.
we can conceive of a presumably infinite number of impossible things.

the fact that they haven't been invented yet would seem to indicate they're impossible.
>>
File: 1488454465703.gif (4MB, 584x900px) Image search: [Google]
1488454465703.gif
4MB, 584x900px
>>8685079
really makes u think
>>
>>8722580
>you're mistaking "conceivable" for "possible."
>we can conceive of a presumably infinite number of impossible things.
I don't think you understand the point of thought experiments. The fact that objects features in the thought experiment don't exist in our world has no bearing on the logic and possible conclusions of the experiment. The experiment is not there to prove or disprove the actual existence of zombies, it's there to challenge and deduce the logic of our notions on whether or not subjective experience can be functionally explained by the same manner the rest of the body's functions are.

>the fact that they haven't been invented yet would seem to indicate they're impossible.
We don't have the technology or knowledge to replicate human brains yet. This is far from a sound argument.
>>
>>8722656
>I don't think you understand the point of thought experiments.
I'm pretty sure you're retarded.
>I can imagine a situation where your hypothesis is false.
>does that situation exist in reality?
>no.
>then fuck off
>>
>>8722656
>We don't have the technology or knowledge to replicate human brains yet.
that is exactly the point.
if the same technology is required to build a p-zombie as is required to build a subject, they are probably the same fucking thing.

a p-zombie cannot exist because an imitation that exact would be the real thing.
>>
>>8685079
I believe the dumbest seeming people are actually the most complete. They may have simple fuckin minds but their minds are fully attached to the time and space they inhabit. Whereas we are constantly at odds with our surroundings, those who just derp through life appearing braindead, they are the most conscious in a way. They own their surroundings in a way that is repulsive but we still envy them
>>
straight up i did a dissertation on AI and consciousness. It messes with you. If you don't believe in some higher power that we have then you can say that a system of pulleys and buckets are conscious. Seeing as our brain is a complex system of inputs and outputs, and we declare ourselves conscious, there is nothing to say that pulleys and strings aren't conscious. UNLESS, you believe that there are varying levels of consciousness, and that you have to reach a certain threshold to be considered conscious - e.g. Monkeys, Dolphins etc. If you want some grade-a stuff read How to create a mind by Ray Kurzweil, or if you're lazy, then the tv show westworld is a great show if you're curious.
>>
>>8722846
>UNLESS, you believe that there are varying levels of consciousness, and that you have to reach a certain threshold to be considered conscious - e.g. Monkeys, Dolphins etc
not coincidentally this is what ~100% of zoologists believe, though they tend to quibble about where that threshold is.
>>
>>8711598
It's difficult to imagine anyone believing anything different as we all call ourselves 'I'. Everything we think we are may be we but everything we direct that bs to is I. We never are I really, we are everything else, but we talk to I.

I doesn't really exist though. And we aren't really all that other bs. So really we don't exist
>>
What is an even harder problem than the hard problem of consciousness? The Identity of subjective experience.

In the game SOMA, a person has a brain scan that and 100 years later we bear witness to the subjective experience of a simulation made by a digital copy of that brain's structure. Having the same memories as the brain that was scanned, it believed that he was in fact still the same person, and must have experienced a seamless "transition", even though 100 years passed. That does paint some questions about the nature of our subjective experience in relation to time and the identity of it and us. What's to say that my subjective experience has been "me" other than right this moment, and the rest is just memories of someone I assume is me but has been experienced by another "subjective entity"? The idea that a subjective experience even has an identity seems messy and kind of absurd. It is generally accepted that if you create a perfect copy of yourself and kill yourself in the exact same moment, the subjective experience of the copy will not be yours. But what is it about "your" experience that really makes it yours, on a technical level? If I were to remove every single atom in my body for a second, and then snap them back at the exact same spot with perfect accuracy, would THAT still be my subjective experience? If the answer to the question is no, then where did it go? What was the system keeping track of the identity of that experience and why it couldn't be me again afterwards?
>>
>>8723101

Would it not make sense that there is no such thing as "your" subjective experience, but rather a unique experience is created every planck second as a function of time, space, and particles, and the only thing that makes it seem continious is the cognitive memory systems of our brain. The only identity is the structure of the brain, while the experience is universal and fundamental. Maybe the identity of the experience becomes somewhat easier to understand if we assume that experience has a physical representation, as photons for light, and the subjective experience at any moment is simply these "experitons". You don't experience anyone else's experitons because you are simply not them, you are your own. Why are these your own? Well maybe since the creation of an experiton is partly a function of space, time, and the structure of other particles around it, you could say that the unique resonance that is created with experitons and your brain, comes together and makes that intrinsic "you" of the subjective experience. You can go on and ask, but why are those the specific experitons that I experience, and it just keeps on getting harder and harder. Maybe those are the experitons that I experience simply because this brain is the only thing in the entire universe that can create subjective experience. :^)
>>
>>8723101
Did you even read the thread? There are already two proposed solutions:
1. The agent does not exist. In order to prove this, it requires explaining the mechanisms of human thought and creating something with the same capabilities, all using our current understanding of biology and physics. If the capabilities can't be replicated, go to 2.
2. Agent is universal. Proving this would require isolating an anomalous mechanic in the human nervous system and demonstrating that it possesses non-local qualities.
>>
>>8723109
Functional explanations seem hopeless since we can't even conceive of how a solution would look like.
>>
>>8682184
I feel like consciousness has only a positive effect in evolution if you have a free will.
Without the free will it wouldn't matter if the data would be processed "in the dark" or not.
>>
>>8723101
I don't think that really poses a problem at all. The digital copy does, indeed, believe that he is the same person as the original once was. And if a person is nothing but an amalgam of memories, personality, thinking patterns, etc.. encoded in the brian, the simulation is indeed the same person, in some sense.
Continuity of identity is a nice abstraction but it only holds up in practice because we don't have the technology to do messed-up shit like brain simulations.
>It is generally accepted that if you create a perfect copy of yourself and kill yourself in the exact same moment, the subjective experience of the copy will not be yours.
The people who generally accept this are simply wrong.
>>
>>8723767
>And if a person is nothing but an amalgam of memories, personality, thinking patterns, etc..

>>It is generally accepted that if you create a perfect copy of yourself and kill yourself in the exact same moment, the subjective experience of the copy will not be yours.
>The people who generally accept this are simply wrong.

We discussed this thought experiment before, and the view that the consciousness won't be yours is incompatible with functionalism.

Because if a person is like you say nothing but an amalgam of memories, personality, thinking patterns, etc, then it follows that everything true about the new person will be true of the old, so you must be the same in every way. If you concede that it still won't be your consciousness, then you're giving room for for consciousness existing in a substrate external to the brain, untouchable by current neurological functional explanation.
>>
>>8723775
>>8723767
Well fuck me, I missed a "not" in the quote and wrote this for nothing. Oh well.
>>
https://www.youtube.com/watch?v=C3aP905vW-c

What did he mean by this?
Thread posts: 305
Thread images: 25


[Boards: 3 / a / aco / adv / an / asp / b / bant / biz / c / can / cgl / ck / cm / co / cock / d / diy / e / fa / fap / fit / fitlit / g / gd / gif / h / hc / his / hm / hr / i / ic / int / jp / k / lgbt / lit / m / mlp / mlpol / mo / mtv / mu / n / news / o / out / outsoc / p / po / pol / qa / qst / r / r9k / s / s4s / sci / soc / sp / spa / t / tg / toy / trash / trv / tv / u / v / vg / vint / vip / vp / vr / w / wg / wsg / wsr / x / y] [Search | Top | Home]

I'm aware that Imgur.com will stop allowing adult images since 15th of May. I'm taking actions to backup as much data as possible.
Read more on this topic here - https://archived.moe/talk/thread/1694/


If you need a post removed click on it's [Report] button and follow the instruction.
DMCA Content Takedown via dmca.com
All images are hosted on imgur.com.
If you like this website please support us by donating with Bitcoins at 16mKtbZiwW52BLkibtCr8jUg2KVUMTxVQ5
All trademarks and copyrights on this page are owned by their respective parties.
Images uploaded are the responsibility of the Poster. Comments are owned by the Poster.
This is a 4chan archive - all of the content originated from that site.
This means that RandomArchive shows their content, archived.
If you need information for a Poster - contact them.